Weak LLN vs Strong LLN












1












$begingroup$


Heres an example of a sequence of r.vs $(X_n)$ that obey the Weak LLN but do not obey the Strong LLN,
$P(X_n=pm n)=1/(2nln n)$ for any $n=1,2dots$ and $P(X_n=0)=1-1/(nln n)$.



I am trying to show why this is so, but I am having trouble with it. We don't have any closed form expressions of the partial sums of $(X_n)$, only the probabilities that each $X_n$ takes at $n$. How do you even compute $sum_n^NX_n/N$?



With regards to the strong Law, it suffices to show that $E[X_1]$ is infinite, but we dont even have that they are identically distributed?



Any help would be greatly appreciated.










share|cite|improve this question











$endgroup$








  • 1




    $begingroup$
    So, what definition of the SLLN do you use?
    $endgroup$
    – Will M.
    Dec 1 '18 at 18:30
















1












$begingroup$


Heres an example of a sequence of r.vs $(X_n)$ that obey the Weak LLN but do not obey the Strong LLN,
$P(X_n=pm n)=1/(2nln n)$ for any $n=1,2dots$ and $P(X_n=0)=1-1/(nln n)$.



I am trying to show why this is so, but I am having trouble with it. We don't have any closed form expressions of the partial sums of $(X_n)$, only the probabilities that each $X_n$ takes at $n$. How do you even compute $sum_n^NX_n/N$?



With regards to the strong Law, it suffices to show that $E[X_1]$ is infinite, but we dont even have that they are identically distributed?



Any help would be greatly appreciated.










share|cite|improve this question











$endgroup$








  • 1




    $begingroup$
    So, what definition of the SLLN do you use?
    $endgroup$
    – Will M.
    Dec 1 '18 at 18:30














1












1








1


2



$begingroup$


Heres an example of a sequence of r.vs $(X_n)$ that obey the Weak LLN but do not obey the Strong LLN,
$P(X_n=pm n)=1/(2nln n)$ for any $n=1,2dots$ and $P(X_n=0)=1-1/(nln n)$.



I am trying to show why this is so, but I am having trouble with it. We don't have any closed form expressions of the partial sums of $(X_n)$, only the probabilities that each $X_n$ takes at $n$. How do you even compute $sum_n^NX_n/N$?



With regards to the strong Law, it suffices to show that $E[X_1]$ is infinite, but we dont even have that they are identically distributed?



Any help would be greatly appreciated.










share|cite|improve this question











$endgroup$




Heres an example of a sequence of r.vs $(X_n)$ that obey the Weak LLN but do not obey the Strong LLN,
$P(X_n=pm n)=1/(2nln n)$ for any $n=1,2dots$ and $P(X_n=0)=1-1/(nln n)$.



I am trying to show why this is so, but I am having trouble with it. We don't have any closed form expressions of the partial sums of $(X_n)$, only the probabilities that each $X_n$ takes at $n$. How do you even compute $sum_n^NX_n/N$?



With regards to the strong Law, it suffices to show that $E[X_1]$ is infinite, but we dont even have that they are identically distributed?



Any help would be greatly appreciated.







probability probability-theory stochastic-processes






share|cite|improve this question















share|cite|improve this question













share|cite|improve this question




share|cite|improve this question








edited Dec 1 '18 at 18:24







user593295

















asked Dec 1 '18 at 18:16









user593295user593295

627




627








  • 1




    $begingroup$
    So, what definition of the SLLN do you use?
    $endgroup$
    – Will M.
    Dec 1 '18 at 18:30














  • 1




    $begingroup$
    So, what definition of the SLLN do you use?
    $endgroup$
    – Will M.
    Dec 1 '18 at 18:30








1




1




$begingroup$
So, what definition of the SLLN do you use?
$endgroup$
– Will M.
Dec 1 '18 at 18:30




$begingroup$
So, what definition of the SLLN do you use?
$endgroup$
– Will M.
Dec 1 '18 at 18:30










1 Answer
1






active

oldest

votes


















1












$begingroup$


Assuming OP means to ask whether or not the random series $Z_N = sumlimits_{n = 2}^N dfrac{X_n}{N}$ converges almost surely. And that OP assumes $(X_n)_{n geq 2}$ is an independent sequence (we also notice $X_1$ makes no sense).




Abridged proof: we fundamentally apply Lindeberg-Feller Central Limit Theorem for triangular array with row-independent random variables (see 8.3 here https://www.ssc.wisc.edu/~xshi/econ715/Lecture_9_local_power.pdf). Our triangular array is $U_{N, k} = dfrac{X_k}{N}.$ Observe $mathbf{E}(U_{N, k}) = 0$ and $mathbf{V}mathrm{ar}(U_{N, k}) = dfrac{k}{N^2 log k}.$ Hence, for $Z_N = sumlimits_{k = 2}^N U_{N, k}$ we have $sigma_N^2 = mathbf{V}mathrm{ar}(Z_N) = sumlimits_{k = 2}^N dfrac{k}{N^2 log k}$ and this series clearly converges to some positive number $sigma^2.$ Finally, the LFCLT holds because
$$sum_{k = 2}^N mathbf{E} left(U_{N, k}^2 mathbf{1}_{left{|U_{n, k}| > frac{varepsilon}{2} sigma right}} right) leq sum_{frac{varepsilon}{2} sigma N leq k leq N} dfrac{k}{N^2 log k} leq dfrac{N(1 - frac{varepsilon}{2} sigma) times N}{N^2 log (frac{varepsilon}{2} sigma N)} to 0.$$ Therefore, $Z_N to mathrm{Norm}(0; sigma^2).$ Q.E.D.



Ammend. What the Lindeberg-Fellet CLT theorem gives is weak convergence. However, we know that convergence almost surely implies weak convergence; in other words, if the random variables $Z_N$ converges almost surely to any limit (which I did not prove), then said limit is normally distributed. Unfortunately,




$Z_N$ will not, in fact, converge almost surely to any limit.




My reasoning follows from the fact that $Z_{N + 1} = dfrac{N}{N+1}Z_N + dfrac{X_{N + 1}}{N + 1}$ so that, assuming $Z_N to Z$ almost surely, for some (finite-valued) limit $Z,$ then $dfrac{X_N}{N} to 0$ almost surely. However, the random variable $dfrac{X_N}{N}$ only has three values: $-1,$ $0$ and $1;$ whence, if the sequence $dfrac{X_N}{N}$ converges to zero, it has to be zero eventually. In other words, what I argumented was that
$$left{ lim_{N to infty} dfrac{X_N}{N} = 0 right} subset bigcup_{N = 1}^infty bigcap_{n = N}^infty {X_n = 0} mathop{=}^{mathrm{def}} bigcup_{N = 1}^infty mathrm{J}_N.$$
By independence, $mathrm{J}_N$ is a null event, hence, except on a null set, no point satisfy $dfrac{X_N}{N} to 0$ and therefore, $Z_N$ does not converge on any subset of positive probability. Q.E.D.






share|cite|improve this answer











$endgroup$













  • $begingroup$
    I think there's a typo in the range of $k$ in the last sum.
    $endgroup$
    – GNUSupporter 8964民主女神 地下教會
    Dec 2 '18 at 4:52










  • $begingroup$
    There was, corrected.
    $endgroup$
    – Will M.
    Dec 2 '18 at 7:53











Your Answer





StackExchange.ifUsing("editor", function () {
return StackExchange.using("mathjaxEditing", function () {
StackExchange.MarkdownEditor.creationCallbacks.add(function (editor, postfix) {
StackExchange.mathjaxEditing.prepareWmdForMathJax(editor, postfix, [["$", "$"], ["\\(","\\)"]]);
});
});
}, "mathjax-editing");

StackExchange.ready(function() {
var channelOptions = {
tags: "".split(" "),
id: "69"
};
initTagRenderer("".split(" "), "".split(" "), channelOptions);

StackExchange.using("externalEditor", function() {
// Have to fire editor after snippets, if snippets enabled
if (StackExchange.settings.snippets.snippetsEnabled) {
StackExchange.using("snippets", function() {
createEditor();
});
}
else {
createEditor();
}
});

function createEditor() {
StackExchange.prepareEditor({
heartbeatType: 'answer',
autoActivateHeartbeat: false,
convertImagesToLinks: true,
noModals: true,
showLowRepImageUploadWarning: true,
reputationToPostImages: 10,
bindNavPrevention: true,
postfix: "",
imageUploader: {
brandingHtml: "Powered by u003ca class="icon-imgur-white" href="https://imgur.com/"u003eu003c/au003e",
contentPolicyHtml: "User contributions licensed under u003ca href="https://creativecommons.org/licenses/by-sa/3.0/"u003ecc by-sa 3.0 with attribution requiredu003c/au003e u003ca href="https://stackoverflow.com/legal/content-policy"u003e(content policy)u003c/au003e",
allowUrls: true
},
noCode: true, onDemand: true,
discardSelector: ".discard-answer"
,immediatelyShowMarkdownHelp:true
});


}
});














draft saved

draft discarded


















StackExchange.ready(
function () {
StackExchange.openid.initPostLogin('.new-post-login', 'https%3a%2f%2fmath.stackexchange.com%2fquestions%2f3021650%2fweak-lln-vs-strong-lln%23new-answer', 'question_page');
}
);

Post as a guest















Required, but never shown

























1 Answer
1






active

oldest

votes








1 Answer
1






active

oldest

votes









active

oldest

votes






active

oldest

votes









1












$begingroup$


Assuming OP means to ask whether or not the random series $Z_N = sumlimits_{n = 2}^N dfrac{X_n}{N}$ converges almost surely. And that OP assumes $(X_n)_{n geq 2}$ is an independent sequence (we also notice $X_1$ makes no sense).




Abridged proof: we fundamentally apply Lindeberg-Feller Central Limit Theorem for triangular array with row-independent random variables (see 8.3 here https://www.ssc.wisc.edu/~xshi/econ715/Lecture_9_local_power.pdf). Our triangular array is $U_{N, k} = dfrac{X_k}{N}.$ Observe $mathbf{E}(U_{N, k}) = 0$ and $mathbf{V}mathrm{ar}(U_{N, k}) = dfrac{k}{N^2 log k}.$ Hence, for $Z_N = sumlimits_{k = 2}^N U_{N, k}$ we have $sigma_N^2 = mathbf{V}mathrm{ar}(Z_N) = sumlimits_{k = 2}^N dfrac{k}{N^2 log k}$ and this series clearly converges to some positive number $sigma^2.$ Finally, the LFCLT holds because
$$sum_{k = 2}^N mathbf{E} left(U_{N, k}^2 mathbf{1}_{left{|U_{n, k}| > frac{varepsilon}{2} sigma right}} right) leq sum_{frac{varepsilon}{2} sigma N leq k leq N} dfrac{k}{N^2 log k} leq dfrac{N(1 - frac{varepsilon}{2} sigma) times N}{N^2 log (frac{varepsilon}{2} sigma N)} to 0.$$ Therefore, $Z_N to mathrm{Norm}(0; sigma^2).$ Q.E.D.



Ammend. What the Lindeberg-Fellet CLT theorem gives is weak convergence. However, we know that convergence almost surely implies weak convergence; in other words, if the random variables $Z_N$ converges almost surely to any limit (which I did not prove), then said limit is normally distributed. Unfortunately,




$Z_N$ will not, in fact, converge almost surely to any limit.




My reasoning follows from the fact that $Z_{N + 1} = dfrac{N}{N+1}Z_N + dfrac{X_{N + 1}}{N + 1}$ so that, assuming $Z_N to Z$ almost surely, for some (finite-valued) limit $Z,$ then $dfrac{X_N}{N} to 0$ almost surely. However, the random variable $dfrac{X_N}{N}$ only has three values: $-1,$ $0$ and $1;$ whence, if the sequence $dfrac{X_N}{N}$ converges to zero, it has to be zero eventually. In other words, what I argumented was that
$$left{ lim_{N to infty} dfrac{X_N}{N} = 0 right} subset bigcup_{N = 1}^infty bigcap_{n = N}^infty {X_n = 0} mathop{=}^{mathrm{def}} bigcup_{N = 1}^infty mathrm{J}_N.$$
By independence, $mathrm{J}_N$ is a null event, hence, except on a null set, no point satisfy $dfrac{X_N}{N} to 0$ and therefore, $Z_N$ does not converge on any subset of positive probability. Q.E.D.






share|cite|improve this answer











$endgroup$













  • $begingroup$
    I think there's a typo in the range of $k$ in the last sum.
    $endgroup$
    – GNUSupporter 8964民主女神 地下教會
    Dec 2 '18 at 4:52










  • $begingroup$
    There was, corrected.
    $endgroup$
    – Will M.
    Dec 2 '18 at 7:53
















1












$begingroup$


Assuming OP means to ask whether or not the random series $Z_N = sumlimits_{n = 2}^N dfrac{X_n}{N}$ converges almost surely. And that OP assumes $(X_n)_{n geq 2}$ is an independent sequence (we also notice $X_1$ makes no sense).




Abridged proof: we fundamentally apply Lindeberg-Feller Central Limit Theorem for triangular array with row-independent random variables (see 8.3 here https://www.ssc.wisc.edu/~xshi/econ715/Lecture_9_local_power.pdf). Our triangular array is $U_{N, k} = dfrac{X_k}{N}.$ Observe $mathbf{E}(U_{N, k}) = 0$ and $mathbf{V}mathrm{ar}(U_{N, k}) = dfrac{k}{N^2 log k}.$ Hence, for $Z_N = sumlimits_{k = 2}^N U_{N, k}$ we have $sigma_N^2 = mathbf{V}mathrm{ar}(Z_N) = sumlimits_{k = 2}^N dfrac{k}{N^2 log k}$ and this series clearly converges to some positive number $sigma^2.$ Finally, the LFCLT holds because
$$sum_{k = 2}^N mathbf{E} left(U_{N, k}^2 mathbf{1}_{left{|U_{n, k}| > frac{varepsilon}{2} sigma right}} right) leq sum_{frac{varepsilon}{2} sigma N leq k leq N} dfrac{k}{N^2 log k} leq dfrac{N(1 - frac{varepsilon}{2} sigma) times N}{N^2 log (frac{varepsilon}{2} sigma N)} to 0.$$ Therefore, $Z_N to mathrm{Norm}(0; sigma^2).$ Q.E.D.



Ammend. What the Lindeberg-Fellet CLT theorem gives is weak convergence. However, we know that convergence almost surely implies weak convergence; in other words, if the random variables $Z_N$ converges almost surely to any limit (which I did not prove), then said limit is normally distributed. Unfortunately,




$Z_N$ will not, in fact, converge almost surely to any limit.




My reasoning follows from the fact that $Z_{N + 1} = dfrac{N}{N+1}Z_N + dfrac{X_{N + 1}}{N + 1}$ so that, assuming $Z_N to Z$ almost surely, for some (finite-valued) limit $Z,$ then $dfrac{X_N}{N} to 0$ almost surely. However, the random variable $dfrac{X_N}{N}$ only has three values: $-1,$ $0$ and $1;$ whence, if the sequence $dfrac{X_N}{N}$ converges to zero, it has to be zero eventually. In other words, what I argumented was that
$$left{ lim_{N to infty} dfrac{X_N}{N} = 0 right} subset bigcup_{N = 1}^infty bigcap_{n = N}^infty {X_n = 0} mathop{=}^{mathrm{def}} bigcup_{N = 1}^infty mathrm{J}_N.$$
By independence, $mathrm{J}_N$ is a null event, hence, except on a null set, no point satisfy $dfrac{X_N}{N} to 0$ and therefore, $Z_N$ does not converge on any subset of positive probability. Q.E.D.






share|cite|improve this answer











$endgroup$













  • $begingroup$
    I think there's a typo in the range of $k$ in the last sum.
    $endgroup$
    – GNUSupporter 8964民主女神 地下教會
    Dec 2 '18 at 4:52










  • $begingroup$
    There was, corrected.
    $endgroup$
    – Will M.
    Dec 2 '18 at 7:53














1












1








1





$begingroup$


Assuming OP means to ask whether or not the random series $Z_N = sumlimits_{n = 2}^N dfrac{X_n}{N}$ converges almost surely. And that OP assumes $(X_n)_{n geq 2}$ is an independent sequence (we also notice $X_1$ makes no sense).




Abridged proof: we fundamentally apply Lindeberg-Feller Central Limit Theorem for triangular array with row-independent random variables (see 8.3 here https://www.ssc.wisc.edu/~xshi/econ715/Lecture_9_local_power.pdf). Our triangular array is $U_{N, k} = dfrac{X_k}{N}.$ Observe $mathbf{E}(U_{N, k}) = 0$ and $mathbf{V}mathrm{ar}(U_{N, k}) = dfrac{k}{N^2 log k}.$ Hence, for $Z_N = sumlimits_{k = 2}^N U_{N, k}$ we have $sigma_N^2 = mathbf{V}mathrm{ar}(Z_N) = sumlimits_{k = 2}^N dfrac{k}{N^2 log k}$ and this series clearly converges to some positive number $sigma^2.$ Finally, the LFCLT holds because
$$sum_{k = 2}^N mathbf{E} left(U_{N, k}^2 mathbf{1}_{left{|U_{n, k}| > frac{varepsilon}{2} sigma right}} right) leq sum_{frac{varepsilon}{2} sigma N leq k leq N} dfrac{k}{N^2 log k} leq dfrac{N(1 - frac{varepsilon}{2} sigma) times N}{N^2 log (frac{varepsilon}{2} sigma N)} to 0.$$ Therefore, $Z_N to mathrm{Norm}(0; sigma^2).$ Q.E.D.



Ammend. What the Lindeberg-Fellet CLT theorem gives is weak convergence. However, we know that convergence almost surely implies weak convergence; in other words, if the random variables $Z_N$ converges almost surely to any limit (which I did not prove), then said limit is normally distributed. Unfortunately,




$Z_N$ will not, in fact, converge almost surely to any limit.




My reasoning follows from the fact that $Z_{N + 1} = dfrac{N}{N+1}Z_N + dfrac{X_{N + 1}}{N + 1}$ so that, assuming $Z_N to Z$ almost surely, for some (finite-valued) limit $Z,$ then $dfrac{X_N}{N} to 0$ almost surely. However, the random variable $dfrac{X_N}{N}$ only has three values: $-1,$ $0$ and $1;$ whence, if the sequence $dfrac{X_N}{N}$ converges to zero, it has to be zero eventually. In other words, what I argumented was that
$$left{ lim_{N to infty} dfrac{X_N}{N} = 0 right} subset bigcup_{N = 1}^infty bigcap_{n = N}^infty {X_n = 0} mathop{=}^{mathrm{def}} bigcup_{N = 1}^infty mathrm{J}_N.$$
By independence, $mathrm{J}_N$ is a null event, hence, except on a null set, no point satisfy $dfrac{X_N}{N} to 0$ and therefore, $Z_N$ does not converge on any subset of positive probability. Q.E.D.






share|cite|improve this answer











$endgroup$




Assuming OP means to ask whether or not the random series $Z_N = sumlimits_{n = 2}^N dfrac{X_n}{N}$ converges almost surely. And that OP assumes $(X_n)_{n geq 2}$ is an independent sequence (we also notice $X_1$ makes no sense).




Abridged proof: we fundamentally apply Lindeberg-Feller Central Limit Theorem for triangular array with row-independent random variables (see 8.3 here https://www.ssc.wisc.edu/~xshi/econ715/Lecture_9_local_power.pdf). Our triangular array is $U_{N, k} = dfrac{X_k}{N}.$ Observe $mathbf{E}(U_{N, k}) = 0$ and $mathbf{V}mathrm{ar}(U_{N, k}) = dfrac{k}{N^2 log k}.$ Hence, for $Z_N = sumlimits_{k = 2}^N U_{N, k}$ we have $sigma_N^2 = mathbf{V}mathrm{ar}(Z_N) = sumlimits_{k = 2}^N dfrac{k}{N^2 log k}$ and this series clearly converges to some positive number $sigma^2.$ Finally, the LFCLT holds because
$$sum_{k = 2}^N mathbf{E} left(U_{N, k}^2 mathbf{1}_{left{|U_{n, k}| > frac{varepsilon}{2} sigma right}} right) leq sum_{frac{varepsilon}{2} sigma N leq k leq N} dfrac{k}{N^2 log k} leq dfrac{N(1 - frac{varepsilon}{2} sigma) times N}{N^2 log (frac{varepsilon}{2} sigma N)} to 0.$$ Therefore, $Z_N to mathrm{Norm}(0; sigma^2).$ Q.E.D.



Ammend. What the Lindeberg-Fellet CLT theorem gives is weak convergence. However, we know that convergence almost surely implies weak convergence; in other words, if the random variables $Z_N$ converges almost surely to any limit (which I did not prove), then said limit is normally distributed. Unfortunately,




$Z_N$ will not, in fact, converge almost surely to any limit.




My reasoning follows from the fact that $Z_{N + 1} = dfrac{N}{N+1}Z_N + dfrac{X_{N + 1}}{N + 1}$ so that, assuming $Z_N to Z$ almost surely, for some (finite-valued) limit $Z,$ then $dfrac{X_N}{N} to 0$ almost surely. However, the random variable $dfrac{X_N}{N}$ only has three values: $-1,$ $0$ and $1;$ whence, if the sequence $dfrac{X_N}{N}$ converges to zero, it has to be zero eventually. In other words, what I argumented was that
$$left{ lim_{N to infty} dfrac{X_N}{N} = 0 right} subset bigcup_{N = 1}^infty bigcap_{n = N}^infty {X_n = 0} mathop{=}^{mathrm{def}} bigcup_{N = 1}^infty mathrm{J}_N.$$
By independence, $mathrm{J}_N$ is a null event, hence, except on a null set, no point satisfy $dfrac{X_N}{N} to 0$ and therefore, $Z_N$ does not converge on any subset of positive probability. Q.E.D.







share|cite|improve this answer














share|cite|improve this answer



share|cite|improve this answer








edited Dec 2 '18 at 20:52

























answered Dec 1 '18 at 21:41









Will M.Will M.

2,445314




2,445314












  • $begingroup$
    I think there's a typo in the range of $k$ in the last sum.
    $endgroup$
    – GNUSupporter 8964民主女神 地下教會
    Dec 2 '18 at 4:52










  • $begingroup$
    There was, corrected.
    $endgroup$
    – Will M.
    Dec 2 '18 at 7:53


















  • $begingroup$
    I think there's a typo in the range of $k$ in the last sum.
    $endgroup$
    – GNUSupporter 8964民主女神 地下教會
    Dec 2 '18 at 4:52










  • $begingroup$
    There was, corrected.
    $endgroup$
    – Will M.
    Dec 2 '18 at 7:53
















$begingroup$
I think there's a typo in the range of $k$ in the last sum.
$endgroup$
– GNUSupporter 8964民主女神 地下教會
Dec 2 '18 at 4:52




$begingroup$
I think there's a typo in the range of $k$ in the last sum.
$endgroup$
– GNUSupporter 8964民主女神 地下教會
Dec 2 '18 at 4:52












$begingroup$
There was, corrected.
$endgroup$
– Will M.
Dec 2 '18 at 7:53




$begingroup$
There was, corrected.
$endgroup$
– Will M.
Dec 2 '18 at 7:53


















draft saved

draft discarded




















































Thanks for contributing an answer to Mathematics Stack Exchange!


  • Please be sure to answer the question. Provide details and share your research!

But avoid



  • Asking for help, clarification, or responding to other answers.

  • Making statements based on opinion; back them up with references or personal experience.


Use MathJax to format equations. MathJax reference.


To learn more, see our tips on writing great answers.




draft saved


draft discarded














StackExchange.ready(
function () {
StackExchange.openid.initPostLogin('.new-post-login', 'https%3a%2f%2fmath.stackexchange.com%2fquestions%2f3021650%2fweak-lln-vs-strong-lln%23new-answer', 'question_page');
}
);

Post as a guest















Required, but never shown





















































Required, but never shown














Required, but never shown












Required, but never shown







Required, but never shown

































Required, but never shown














Required, but never shown












Required, but never shown







Required, but never shown







Popular posts from this blog

Quarter-circle Tiles

build a pushdown automaton that recognizes the reverse language of a given pushdown automaton?

Mont Emei